Added topic tags to some Y&F-12 problems
[course.git] / latex / problems / Young_and_Freedman_12 / problem27.30.tex
1 \begin{problem*}{27.30} % Lorentz force
2 A particle with initial velocity $\vect{v}_0=5.85\E{3}\U{m/s}\jhat$
3 enters a region of uniform electric and magnetic fields.  The magnetic
4 field in the region is $\vect{B}=-(1.35\U{T})\khat$.  Calculate the
5 magnitude and direction of the electric field in the region if the
6 particle is to pass through undeflected, for a particle of
7 charge \Part{a} $+0.640\U{nC}$ and \Part{b} $-0.640\U{nC}$.  You can
8 ignore the weight of the particle.
9 \end{problem*}
10
11 \begin{solution}
12 \Part{a}
13 This is a straigtforward application of the Lorentz force.
14 \begin{align}
15   \vect{F} &= 0 = q(\vect{E} + \vect{v}\times\vect{B}) \\
16   \vect{E} &= -\vect{v}\times\vect{B}
17     = 5.85\E{3}\U{m/s}\cdot1.35\U{T}(\jhat \times \khat)
18     = \ans{7.90\U{V/m}\ihat} \;.
19 \end{align}
20
21 \Part{b}
22 Because the charge canceled out in \Part{a}, the electric field should
23 be the same: $\vect{E}=\ans{7.90\U{V/m}\ihat}$.
24 \end{solution}